Đến nội dung

nguyentrungphuc26041999 nội dung

Có 382 mục bởi nguyentrungphuc26041999 (Tìm giới hạn từ 29-04-2020)



Sắp theo                Sắp xếp  

#482580 $\boxed{\text{Chuyên Đề}}$ Bất đẳng thức - Cực trị

Đã gửi bởi nguyentrungphuc26041999 on 11-02-2014 - 20:10 trong Bất đẳng thức và cực trị

28) Cho $a;b;c>0$ thỏa $a+b+c=3$. Cmr: $\sum \frac{a+1}{b^2+1}\geq 3$

làm thế này không biết có sai không

ta có $ab+bc+ca\leq 3$

$\Rightarrow \sum \frac{a+1}{b^{2}+1}\geq \sum \frac{a+1}{b^{2}+ab+bc+ca}=\sum \frac{a+1}{\left ( b+a \right )\left ( b+c \right )}$

đặt $\left\{\begin{matrix} a+b=x & \\ b+c=y & \\ c+a=z & \end{matrix}\right.$

$\sum \frac{2x+2z-y}{3xy}\geq 3$ với $x+y+z=6$

đến đoạn này việc đánh giá là hoàn toàn đơn giản chỉ có đièu mình sai hệ số chỗ nào không phát hiện ra,mong các bạn sửa hộ




#482573 $\left ( x^{4}+1 \right )\left ( y^{4...

Đã gửi bởi nguyentrungphuc26041999 on 11-02-2014 - 19:50 trong Bất đẳng thức và cực trị

cho $x,y$ dương thoả mãn $x+y=\sqrt{10}$

tìm MIN $\left ( x^{4}+1 \right )\left ( y^{4}+1 \right )$




#481644 Trận 2 - PT, HPT

Đã gửi bởi nguyentrungphuc26041999 on 07-02-2014 - 17:11 trong Thi giải toán Marathon cấp THCS 2014

Hai toán thủ nguyentrungphuc26041999  và  Frankie nole có cùng địa chỉ IP và có lời giải giống hệt nhau. Điều này là gian lận.

 

BTC đã chấm điểm cho nguyentrungphuc26041999 và cảnh cáo toán thủ này. 

 

Toán thủ Frankie nole bị loại vì hành vi gian lận

xin lỗi BTC nhưng hôm đó,mạng nhà em bị rớt,mất mạng vài tuần nên đến nhà toán thủ frankynole để làm,do bài này khá dễ nên làm giống nhau là chuyện bình thường vì đây là cách mà phần lớn các toán thủ đều làm.




#481284 $a^3+b^3+c^3+ab^2+bc^2+ca^2\geq a^2b+b^2c+c^2a+3$

Đã gửi bởi nguyentrungphuc26041999 on 05-02-2014 - 22:49 trong Bất đẳng thức và cực trị

 

$a^{3}+ab^{2}\geq 2a^{2}b$

$\Rightarrow \sum a^{3}+\sum ab^{2}\geq \sum 2a^{2}b$

$\sum a^{3}+\sum ab^{2}+ \sum a^{2}b$

$\sum a^{3}+\sum a^{2}c+ \sum a^{2}b= 3\sum a^{2}\geq 9$

$\Rightarrow 3\frac{\sum a^{3}+\sum ab^{2}}{2}\geq 9$

$\Rightarrow \sum a^{3}+\sum ab^{2}\geq 6$

$\Rightarrow \sum a^{3}+\sum ab^{2}\geq \sum a^{2}b+3$

nói rõ hơn cái,chú làm tắt quá




#481269 Giải phương trình nghiệm nguyên dương: $x^{2}-2y^{2}...

Đã gửi bởi nguyentrungphuc26041999 on 05-02-2014 - 22:09 trong Đại số

Mình có bài này mong các bạn giải giùm:

Giải phương trình nghiệm nguyên dương

$x^{2}-2y^{2}=5$

Mình đang học lớp 8 nhé.

Thanks.

$x$ lẻ,$y$ chẵn

ta có $x^{2}-1-2y^{2}=4$

$\Leftrightarrow \left ( x-1 \right )\left ( x+1 \right )+2y^{2}=4$

$\left ( x-1 \right )\left ( x+1 \right )\vdots 8$

$\Rightarrow VT\vdots 8$

suy ra vô lý




#480879 $CMR \frac{a+1}{b^2+1}+\frac{b+1...

Đã gửi bởi nguyentrungphuc26041999 on 04-02-2014 - 18:07 trong Bất đẳng thức và cực trị

ta có 

$ab+bc+ca\leq \frac{\left ( a+b+c \right )^{2}}{3}=3$

$\Rightarrow \frac{a+1}{b^{2}+1}\geq \frac{2a+b+c}{\left ( b+c \right )\left ( b+a \right )}$

đặt $\left\{\begin{matrix} a+b=x & \\ a+c=y & \\ b+c=z & \end{matrix}\right.$

cần chứng minh $\frac{1}{x}+\frac{1}{y}+\frac{1}{z}+\sum \frac{x}{yz}\geq 3$ với $x+y+z= 6$

dễ rồi




#479104 Trận 2 - PT, HPT

Đã gửi bởi nguyentrungphuc26041999 on 26-01-2014 - 09:38 trong Thi giải toán Marathon cấp THCS 2014

hệ tương đương với $\left\{\begin{matrix} 2x^{2}-5xy+3y^{2}=0 & \\ 4x^{2}-6x+1=y^{2}-3y & \end{matrix}\right.$

$\Leftrightarrow \left\{\begin{matrix} \left ( x-y \right )\left ( 2x-3y \right )=0 & \\ 4x^{2}-6x+1=y^{2}-3y & \end{matrix}\right.$

trường hợp 1 nếu $x=y$

hệ trở thành $\left\{\begin{matrix} x=y & \\ 3x^{2}-3x +1=0& \end{matrix}\right.$

$\Delta =-3< 0$ nên hệ vô nghiệm

trường hợp 2 

nếu $2x=3y$

hệ trở thành $\left\{\begin{matrix} 2x=3y & \\ 8y^{2}-6y+1=0 & \end{matrix}\right.$

$\left\{\begin{matrix} 2x=3y & \\ \left ( 4y-1 \right )\left ( 2y-1 \right )=0 & \end{matrix}\right.$

nếu $y=\frac{1}{4}$ $\Rightarrow x=\frac{3}{8}$

 nếu $y=\frac{1}{2}$ $\Rightarrow x=\frac{3}{4}$

Vậy $\left ( x,y \right )=\left \{ \left ( \frac{3}{8},\frac{1}{4} \right ),\left ( \frac{3}{4},\frac{1}{2} \right ) \right \}$

 

d = 10

S = 36




#478135 Cho $x+y+z=xyz$

Đã gửi bởi nguyentrungphuc26041999 on 19-01-2014 - 22:10 trong Bất đẳng thức và cực trị

Cho $x,y,z>0$ thỏa mãn $x+y+z=xyz$

Tìm Max  $\frac{2}{\sqrt{x^2+1}}+\frac{1}{\sqrt{y^2+1}}+\frac{1}{\sqrt{z^2+1}}$

Đặt $x=\frac{1}{a},y=\frac{1}{b},z=\frac{1}{c}$

ta có $ab+bc+ca=1$

trên đề 

$\frac{2}{\sqrt{\frac{1}{a^{2}}+1}}+\frac{1}{\sqrt{\frac{1}{b^{2}}+1}}+\frac{1}{\sqrt{\frac{1}{c^{2}}+1}}=\frac{2a}{\sqrt{a^{2}+ab+bc+ca}}+\frac{b}{\sqrt{b^{2}+ab+bc+ca}}+\frac{c}{\sqrt{c^{2}+ab+bc+ca}}$

$\frac{2a}{\sqrt{\left ( a+b \right )\left ( a+c \right )}}+\frac{2b}{\sqrt{4\left ( b+c \right )\left ( b+a \right )}}+\frac{2c}{\sqrt{4\left ( c+a \right )\left ( c+b \right )}}\leq \frac{a}{a+b}+\frac{a}{a+c}+\frac{b}{b+a}+\frac{b}{4\left ( b+c \right )}+\frac{c}{a+c}+\frac{c}{4\left ( b+c \right )}=\frac{9}{4}$




#478127 $\sum \frac{1}{1+a}\geq \sum...

Đã gửi bởi nguyentrungphuc26041999 on 19-01-2014 - 21:46 trong Bất đẳng thức và cực trị



Áp dụng BĐT $\frac{1}{1+a}+\frac{1}{1+b}\geq \frac{2}{1+\sqrt{ab}}$

=> VT $\geq \sum \frac{1}{1+\sqrt{ab}}=\sum \frac{1}{\sqrt[4]{a^{2}b^{2}}+1}\geq \sum \frac{1}{1+\sqrt[4]{ab^{3}}}$

( ta có $ab^{3}+bc^{3}+ca^{3}\geq a^{2}b^{2}+b^{2}c^{2}+c^{2}a^{2}$ nên BĐT đúng)

chỗ này không ổn lắm 

mình nghĩ là thế này

sử dụng bất đẳng thức $\frac{1}{1+a}+\frac{1}{1+b}\geq \frac{2}{1+\sqrt{ab}}$

ta có 

$\frac{1}{1+a}+\frac{1}{1+b}+\frac{2}{1+a}\geq \frac{2}{1+\sqrt{ab}}+\frac{2}{1+\sqrt{a^{2}}}\geq \frac{4}{1+\sqrt[4]{a^{3}b}}$

thiết lập các bất đẳng thức tương tự cộng lại ta được điều phải chứng minh




#478125 cho a,b,c là các số dương. và a+b+c=1

Đã gửi bởi nguyentrungphuc26041999 on 19-01-2014 - 21:42 trong Bất đẳng thức và cực trị

nếu bạn suy ra abc$\leq \frac{1}{27}$ thì dấu bằng xảy ra khi a=b=c=$\frac{1}{3}$ . tức là min ra -8/9

sao lại thế nếu $a=b=c=\frac{1}{3}$ thì $P=8$

đúng chứ




#478053 cho a,b,c là các số dương. và a+b+c=1

Đã gửi bởi nguyentrungphuc26041999 on 19-01-2014 - 15:40 trong Bất đẳng thức và cực trị



cho a,b,c là các số dương. và a+b+c=1. Tìm Min của biểu thức P=$\sqrt[3]{\left ( \frac{1}{ab} -1\right )\left ( \frac{1}{bc}-1 \right )\left ( \frac{1}{ca}-1 \right )}$

Từ giả thiết suy ra  $abc\leq \frac{1}{27}$

xét biểu thức trong căn sau khi quy đồng và làm gọn ta được 

$\frac{1-ab-bc-ca+abc\left ( a+b+c \right )-a^{2}b^{2}c^{2}}{a^{2}b^{2}c^{2}}=\frac{1-ab-bc-ca}{a^{2}b^{2}c^{2}}+\frac{1}{abc}+1\geq \frac{1-\frac{1}{3}}{\frac{1}{27^{2}}}+\frac{1}{\frac{1}{27}}-1$

$=512$

$\Rightarrow P\geq \sqrt[3]{512}=8$

không biết tính có sai không mà thấy số lẻ quá




#478037 Cho các số dương $a,b,c$ thỏa mãn $(a^2+1)(b^2+1)(c^2+1)\...

Đã gửi bởi nguyentrungphuc26041999 on 19-01-2014 - 15:08 trong Bất đẳng thức và cực trị

Cho các số dương $a,b,c$ thỏa mãn $(a^2+1)(b^2+1)(c^2+1)\leq8$. CMR:$$ab+bc+ca+abc\leq4$$

đây nè 

dễ chứng minh $a^{2}+b^{2}+c^{2}\geq 3$

ta có $8=1+a^{2}+b^{2}+c^{2}+\sum a^{2}b^{2}+a^{2}b^{2}c^{2}\geq \sum a^{2}b^{2}+a^{2}b^{2}c^{2}+4$

$8\geq \sum a^{2}b^{2}+a^{2}b^{2}c^{2}+4=\sum \left ( a^{2}b^{2}+1 \right )+a^{2}b^{2}c^{2}+1\geq 2\left ( ab+bc+ca \right )$

$\Rightarrow ab+bc+ca+abc\leq 4$




#478033 $\left ( x^{4}+1 \right )\left ( y^{4...

Đã gửi bởi nguyentrungphuc26041999 on 19-01-2014 - 15:03 trong Bất đẳng thức và cực trị

Cho $x,y$ là các số thực dương và  $x+y=k$ $\left ( k>0 \right )$

tìm MIN $\left ( x^{4}+1 \right )\left ( y^{4}+1 \right )$




#478006 Đề thi chọn học sinh giỏi thành phố Biên Hòa 2013 - 2014 ( Đồng Nai )

Đã gửi bởi nguyentrungphuc26041999 on 19-01-2014 - 11:43 trong Tài liệu - Đề thi

                                         THI CHỌN HỌC SINH GIỎI CÂP THÀNH PHỐ
                                                        NĂM HỌC 2013 - 2014
                                               
     Ngày 18 tháng 01 năm 2014  

Câu 3 :

Cho 3 số $a;b;c$ thỏa : $\left\{\begin{matrix} a^{2}+b^2+c^2=1 & \\ a^3+b^3+c^3=1 & \end{matrix}\right.$
Tính : $S=a+b^2+c^2$

Ta có $1\leq a,b,c\leq -1$

trừ vế với vế của phương trình ta được $a^{2}\left ( 1-a \right )+b^{2}\left ( 1-b \right )+c^{2}\left ( 1-c \right )=0$

mà $1-a\geq 0,1-b\geq 0,1-c\geq 0$

dấu bằng xảy ra 1 trong 3 số bằng 1 




#477541 toán bất đẳng thức

Đã gửi bởi nguyentrungphuc26041999 on 16-01-2014 - 16:17 trong Bất đẳng thức và cực trị

mấy bác giải giùm em với !!!!

 

1/Cho a,b,c >0 và a+b+c=6abc. chứng minh rằng:

$\frac{bc}{a^3(c+2b)}+\frac{ac}{b^3(a+2c)}+\frac{ab}{c^3(b+2a)}$$\geq 2$

 

 

2/Cho $\alpha ,\beta ,\gamma \geq 0$. tìm min:

M= $\frac{\alpha x}{y+z}+\frac{\beta y}{x+z}+\frac{\gamma z}{y+x}$

 

(x,y,z>0)

 

thank các bác. em làm lính mới chưa bít cách đăng bài 

1 đặt $\left\{\begin{matrix} \frac{1}{a}=x & \\ \frac{1}{b}=y & \\ \frac{1}{c}=z & \end{matrix}\right.$

khi đó ta có 

$xy+yz+zx=6$

bất đẳng thức tương đương$\sum \frac{x^{3}}{y+z}\geq 2$

theo bất đẳng thức BCS 

$\sum \frac{x^{3}}{y+z}=\sum \frac{x^{4}}{2xy+xz}\geq \frac{\left ( x^{2}+y^{2}+z^{2} \right )^{2}}{3\left ( xy+yz+zx \right )}\geq \frac{\left ( xy+yz+zx \right )^{2}}{3\left ( xy+yz+zx \right )}=2$

dấu bằng tự tìm




#477538 Chứng minh $a^{4}+b^{4}+c^{4}+abc(a+b+c)...

Đã gửi bởi nguyentrungphuc26041999 on 16-01-2014 - 16:01 trong Bất đẳng thức và cực trị

Mình có bài này mong các bạn giải giùm:

Chứng minh:

$a^{4}+b^{4}+c^{4}+abc(a+b+c)\geq \frac{2}{3}(ab+bc+ca)$

Mình đang học lớp 8 nhé.

Thanks

có đúng đề không,2 bên không đồng bậc




#475774 chứng minh $x^2y^2(x^2+y^2)\leq 2$

Đã gửi bởi nguyentrungphuc26041999 on 06-01-2014 - 20:05 trong Bất đẳng thức và cực trị

cho 2 số dương x,y thỏa mãn $x+y=2$

c/m $x^2y^2(x^2+y^2)$

ta có 

$\frac{1}{2}xy.2xy.\left ( x^{2}+y^{2} \right )\leq \frac{1}{2}\frac{\left ( x+y \right )^{2}}{4}.\frac{\left ( x^{2}+y^{2}+2xy \right )^{2}}{4}= 2$

dấu bằng xảy ra khi x=y=1




#475771 Trận 1 - Phương trình nghiệm nguyên ...

Đã gửi bởi nguyentrungphuc26041999 on 06-01-2014 - 19:57 trong Thi giải toán Marathon cấp THCS 2014

chết thật,sót mất $m=0$ loại

ghi nhầm $n$ chẵn lẻ nữa




#475131 Trận 1 - Phương trình nghiệm nguyên ...

Đã gửi bởi nguyentrungphuc26041999 on 03-01-2014 - 23:00 trong Thi giải toán Marathon cấp THCS 2014

Giả sử $m\neq 1$

$\Rightarrow m> 1$

Khi $n$ lẻ $\Rightarrow n^{2}+1\vdots 2$

$\Rightarrow \left ( n^{2}+1 \right )^{2^{k}}\equiv 1\left ( mod4 \right )$

$\Rightarrow \left ( 44n^{3}+11n^{2}+10n+2 \right )\equiv 2\left ( mod4 \right )$

$\Rightarrow VT\equiv 2\left ( mod4 \right )$

$\Rightarrow N\vdots 2$

$\Rightarrow N^{m}\vdots 4$

Suy ra vô lý 

khi n chẵn,$44n^{3}+11n^{2}+10n+2\vdots 2$

$N\vdots 2$

đặt $N=2^{a}b$ với $b\equiv 1\left ( mod2 \right )$ 

đặt $n^{2}+1=2c$ với $c\equiv 1\left ( mod2 \right )$

Phương trình trở thành 

$\left ( 2c \right )^{2^{k}}.\left ( 44n^{3}+11n^{2}+10n+2 \right )=\left ( 2^{a}b \right )^{m}$

$2^{2^{k}}c^{2^{k}}.\left ( 44n^{3}+11n^{2}+10n+2 \right )= 2^{am}b^{m}$

do $b$ lẻ  nên $2^{k}=am$

$\Rightarrow m$ chẵn

$\left ( 44n^{3}+11n^{2}+10n+2 \right )$  là số chính phương 

$44n^{3}+11n^{2}+10n+2=44n^{3}+12n^{2}+8n-\left ( n^{2}-2n-2 \right )\equiv -n^{2}+2n+2\left ( mod4 \right )$

$\Rightarrow -n^{2}+2n+2=3-\left ( n-1 \right )^{2}$

nếu $\left ( n-1 \right )^{2}\equiv 1\left ( mod 4 \right )$

$\Rightarrow 3-\left ( n-1 \right )^{2}\equiv 2\left ( mod 4 \right )$

suy ra $44n^{3}+11n^{2}+10n+2$ không phải số chính phương 

nếu $\left ( n-1 \right )^{2}\equiv 0\left ( mod4 \right )$

$\Rightarrow 3-\left ( n-1 \right )^{2}\equiv 3\left ( mod4 \right )$

suy ra $44n^{3}+11n^{2}+10n+2$ không là số chính phương 

Suy ra $m=1$

 

Điểm bài :6đ ( chổ màu đỏ xem lại cách lập luận)

Nhầm lẫn nghiêm trọng giữa hai trường hợp $n$ lẻ và $n$ chẵn.

S = 16,3 + 6*3 = 34.3




#475037 Tìm min của $A={{x}^{2}}+{{y...

Đã gửi bởi nguyentrungphuc26041999 on 03-01-2014 - 19:51 trong Bất đẳng thức và cực trị

Sai nhiều lắm bạn à.

$\left ( x+\frac{1}{2}y \right )^{2}-(x-\frac{1}{2}y)+\frac{1}{4}$

không thể nhóm $(x+\frac{1}{2}y)^2$ và $(x-\frac{1}{2}y)$

phân tích các  $y^2$ thì thừa $\frac{1}{4}y^2$

Và một vài chỗ khác

fix rồi




#474934 Tìm min của $A={{x}^{2}}+{{y...

Đã gửi bởi nguyentrungphuc26041999 on 03-01-2014 - 11:09 trong Bất đẳng thức và cực trị

Tìm min của $A={{x}^{2}}+{{y}^{2}}-xy-x+y+1$

không biết mình phân tích có sai không được kết quả khác 

$A=\left ( x-\frac{1}{2}y \right )^{2}-(x-\frac{1}{2}y)+\frac{1}{4}+y^{2}+\frac{1}{2}y+\frac{1}{16}+\frac{11}{16}$

$A=\left ( x-\frac{1}{2y}-1\frac{1}{2} \right )^{2}+\left ( y+\frac{1}{4} \right )^{2}+\frac{11}{16}\geq \frac{11}{16}$




#474473 $\frac{1}{1+a^{2}}+\frac{1...

Đã gửi bởi nguyentrungphuc26041999 on 01-01-2014 - 16:06 trong Bất đẳng thức và cực trị

chuyển vế,biến đổi tương đương ta được 

$\frac{\left ( x-y \right )^{2}\left ( xy-1 \right )}{\left ( 1+x^{2} \right )\left ( 1+y^{2} \right )\left ( 1+xy \right )}\geq 0$

hiển nhiên đúng




#474167 $\sqrt{2}(a+b+c)\leq \sqrt{a^2+b^2}+...

Đã gửi bởi nguyentrungphuc26041999 on 31-12-2013 - 16:31 trong Bất đẳng thức và cực trị

cho$a,b,c> o$. c/m:

$\sqrt{2}(a+b+c)\leq \sqrt{a^2+b^2}+\sqrt{c^2+b^2}+\sqrt{a^2+c^2}$ 

 

$\sqrt{2}(a+b)= \sqrt{2(a+b)^{2}}$$\leq \sqrt{4(a^{2}+b^{2})}$$= 2\sqrt{a^{2}+b^{2}}$

Tương tự ta có $\sqrt{2}(b+c)\leq 2\sqrt{b^{2}+c^{2}}$

$\sqrt{2}(a+c)\leq 2\sqrt{a^{2}+c^{2}}$

cộng các vế ta có đpcm

thế này nhìn thuận mắt hơn tí

$\sqrt{2\left ( a^{2}+b^{2} \right )}+\sqrt{2\left ( b^{2}+c^{2} \right )}+\sqrt{2\left ( c^{2}+a^{2} \right )}\geq 2\left ( a+b+c \right )$

theo scharwt $\sqrt{2\left ( a^{2}+b^{2} \right )}\geq a+b$

thiết lập mấy cái tương tự cộng lại là xong




#472512 Chứng minh: $\sum \frac{a}{3a+b+c}\le...

Đã gửi bởi nguyentrungphuc26041999 on 23-12-2013 - 19:53 trong Bất đẳng thức và cực trị

Cho $a,b,c>0.$ Chứng minh $A=\frac{a}{3a+b+c}+\frac{b}{3b+a+c}+\frac{c}{3c+b+a}\leqslant \frac{3}{5}.$

theo cachy schawrt

$a\left ( \frac{4}{2a}+\frac{9}{a+b+c} \right )\geq \frac{25a}{3a+b+c}$

thiết lập các bất đẳng thức tương tự cộng lại ta có đpcm




#472457 Cho (O;R)

Đã gửi bởi nguyentrungphuc26041999 on 23-12-2013 - 15:31 trong Hình học

Cho $(O;R)$ , A chuyển động trên $(O)$, B cố định nằm trong đường tròn và khác O. $AB$ cắt đtròn tại C. Vẽ $(I)$ đkính $OC$ cắt $AB$ tại D.Vẽ tiếp tuyến của $(O)$ tại A cắt $OD$ tại M.Tìm quỹ tích M(ko cần phần đảo)

gợi ý: $K$ laf điểm nằm trên $OB$ sao cho $OK=R^{2}/OB$ $M$ chạy trên đường thẳng vuông góc với $OK$ tại $K$